A spring with a spring constant of 200 is compressed 0.5 . What is the potential energy stored in the spring

Answers

Answer 1

Answer:

E = 25 J

Explanation:

Given that,

Spring constant, k = 200

Compression, x = 0.5

We need to find the potential energy stored in the spring. The formula for the stored potential energy in the spring is given by :

[tex]E=\dfrac{1}{2}kx^2[/tex]

Put all the values,

[tex]E=\dfrac{1}{2}\times 200\times 0.5^2\\\\E=25\ J[/tex]

So, 25 J of potential enersy is stored in the spring.


Related Questions

cuales son las características de un generador eléctrico? ​

Answers

Answer:

Los generadores eléctricos funcionan según el principio de inducción electromagnética. Una bobina conductora (una bobina de cobre enrollada firmemente sobre un núcleo de metal) se hace girar rápidamente entre los polos de un imán tipo herradura. La bobina conductora junto con su núcleo se conoce como armadura.

Snell's Law: Light traveling through water comes to a glass surface at an angle of incidence of
48.5 degrees. If the angle of refraction is 38.5 degrees, what is the index of refraction of the
glass?

Answers

Answer:

1. The best definition of refraction is ____.

a. passing through a boundary

b. bouncing off a boundary

c. changing speed at a boundary

d. changing direction when crossing a boundary

 

Answer: D

Bouncing off a boundary (choice b) is reflection. Refraction involves passing through a boundary (choice a) and changing speed (choice c); however, a light ray can exhibit both of these behaviors without undergoing refraction (for instance, if it approaches the boundary along the normal). Refraction of light must involve a change in direction; the path must be altered at the boundary.

The diagram shows a motion map for a train. A motion map. The position line is a long black arrow pointing right that is labeled x. Above the line are three dots with a vector pointing away from x that start farther right from x. There are three black dots that are in a vertical column above the point of the third vector. There are two yellow vectors with no black dots pointed toward x starting just above the vertical column of three dots. Which statement is supported by the motion map? The train traveled south and then north. The train moved at a velocity of 75 mph north. The train had a greater velocity when it moved away from the origin. The train stopped for 3 seconds.

Answers

Answer:

The Train Stopped For 3 Seconds

Explanation:

Just did quiz on edg its right

According to the question, the train stopped for 3 seconds. So the correct option is D.

What is a motion map?

The one-dimensional plots or graphs are constructed from a position line. It is a horizontal line, along which the position of an object can be indicated, while also indicating its acceleration and velocity.

Arrows are used for marking acceleration and velocity and the direction of these arrows marks the direction of the acceleration or the velocity. The size of the arrow is used to represent the magnitude of the velocity or the acceleration.

Thus, to observe the position, acceleration and velocity of an object in motion, a motion map is used.

Velocity and acceleration vector quantities have magnitude as well as direction. Velocity and acceleration are functions of displacement which is the distance covered in a particular direction. Unlike speed which is a function of distance. Distance just measures the distance covered.

Therefore, according to the question, the correct option is D.

Read more about motion maps, here

https://brainly.com/question/13987788

#SPJ5

A spring has an unstretched length of 0.2 m. A 0.2 kg mass hanging from the spring stretches it to an equilibrium length of 0.3 m. Suppose the mass is pulled down to where the spring’s length is 0.5 m. When it is released, it begins
to oscillate. What is the amplitude of the oscillation?

Answers

I think it is c
But there could be some other people that think differently

help for brainliest award​

Answers

Answer:

a) actual reading=2mm+(0.1×8)=2.8 mm=0.28 cm

b)actual reading=5 mm+(0.1×7)=5.7mm=0.57 cm

c)actual reading=9 mm=0.9 cm

d)actual reading=0 mm+(0.1×7)=0.7 mm=0.07 cm

e)actual reading=19 mm+(0.1×9)=19.9 mm=1.99 cm

f)actual reading=15 mm+(0.1×3)=15.3 mm=1.53 cm

g)actual reading=11 mm(0.1×5)=11.5 mm=1.15 cm

Which of the following variables are conserved?
a. Velocity
b. Acceleration
c. Momentum
d. Displacement

Answers

Answer: C

Explanation:

A mass hanging from a pendulum at the end of a string of negligible mass. The string has a length of 0.84 m. With the string hanging vertically, the object is given an initial velocity of 1.4 m/s parallel to the ground and swings upward on a circular arc. Eventually, the object comes to a momentary halt at a point where the string makes an angle θ with its initial vertical orientation and then swings back downward. Find the angle θ.

Answers

Explanation:

tan(theta)=(v^2)/(length of string × g)

=(1.4^2)/(0.84 × 9.8m/s )

=1.96/(8.232)

=0.2380952381

theta=arctan(0.2380952381)

=13.4°

c) What's meant by:
The electromotive force of an electric cell - 1.5 volt.

Answers

Answer:

The electromotive force is the voltage measured across the cell terminals when "NO" current is being drawn. That is why electromotive forces are used in Wheatstone Bridges which measure the resistance of an external object.

Just measuring the voltage across a cell does not give one the true EMF because current drawn from the cell will cause a reduction of voltage of I * R where R is the internal resistance of the cell.

A cyclist is moving at a speed of 15 m/s. If the combined mass of the bike and person is 100 kg,
what is the momentum of the cyclist/bike?

Answers

Answer:

Explanation:

Momentum is equal to mass times velocity in kg and m/s, respectively. Therefore,

p = 100(15) so

p = 1500 [tex]\frac{kg*m}{s}[/tex]

1. The first thing to do in the event of brake failure is to a. use the parking brake to stop your carb. pump the brake pedal rapidlyc. shift to a lower geard. steer against the curb 2. Engine failure can be caused by a. a broken timing gearb. a lack of fuelc. extreme heatd. all of the above 3. If your engine stalls a. your power brakes won't work at allb. your power steering won't work at allc. your power brakes and power steering won't work very welld. you should pump your power brakes 4. If your engine is flooded, you will probably a. smell gasolineb. see steam coming out from under the hoodc. have your engine stall after going through a large puddle of waterd. all of the above 5. The most common kind of steering failure is a. total system failureb. power-assist failurec. both a

Answers

Answer:

The correct answers are -

1. b. pump the brake pedal rapidly

2. d. all of the above

3. c. your power brakes and power steering won't work very well

4. a. gasoline

5. b. power-assist failure

Explanation:

Brakes help in stopping the vehicle by causing resistance to wheels. IF brake failure it is advised to pump the breaks rapidly in order to cause resistance. Engine failure can be caused by various reasons such as a broken timing gear, no fuel, or getting extremely heated.

In the case of stalling of engine most common effects as power steering not working properly or power brakes not working well. The flooded engine means the engine has too much gasoline that it wont starts.

If you were testing materials to see whether they conduct electricity using a circuit and bulbs, what variable would you be changing?

A. The strength of the battery
B. The number of bulbs
C. The material
D. The time it takes the electricity to complete the circuit

Answers

Answer: C. The material

Explanation:

In order to check whether a material conducts electricity, the first thing to do would be to design a circuit. Connect a positive wire to the batteries and then another wire from the battery to the bulb and then a wire from the bulb as well.

Then bring different materials to test for conductivity by connecting the wires to the material. If the bulb lights then it is a conductor and if it doesn't the material is not a conductor. The material is therefore the changing variable.

A cable car is being pulled up a mountain at 7.5 meters per second. Usually, the car takes 120 seconds to move all the way up the mountain. If its velocity stays constant, how much time will it take the cable car to move 39 meters?

Answers

Answer:

5.2 s

Explanation:

From the question given above, the following data were obtained:

Velocity = 7.5 m/s

Displacement = 39 m up

Time =?

The time taken for the cable cab to get to 39 m can be obtained as follow:

Velocity = Displacement / Time

7.5 = 39 / time

Cross multiply

7.5 × time = 39

Divide both side by 7.5

Time = 39 / 7.5

Time = 5.2 s

Thus, the time taken for the cable cab to get to 39 m up is 5.2 s

Bài 1. Ở nhiệt độ 17°C, có bao nhiêu phần trăm phân tử khí có các vận tốc sai khác
không quá 0,5m/s, các vận tốc sau đây? Lấy g=0,029kg / mol
a. v=v; b. v=0.lv

Answers

Answer:

hi i only dont understand want ur saying

difference between liquid solid and gas in the arrangement of molecules​

Answers

Answer:

Note that:

Particles in a:

gas are well separated with no regular arrangement.

liquid are close together with no regular arrangement.

solid are tightly packed, usually in a regular pattern.

Explanation:

The difference is in the movement in the molecules and their spatial arrangement.Molecules in gaseous states have the biggest freedom of movement, which means that they can move freely around. This is mainly due to the fact that intermolecular forces between the molecules are not as strong as in other states.


that’s the answer i got when i looked it up

The energy of an electromagnetic wave changes proportionally to which

other property?

O A Speed

O B. Shift

O c. Wavelength

O D. Frequency

Answers

Answer:

D. Frequency

Explanation:

The energy of an  electromagnetic wave is proportional to frequency, mathematically it is expressed as;

E ∝ f

E = hf

where;

h is Planck's constant = 6.626 x 10⁻³⁴ Js

The equation above can also be expanded to;

[tex]E = hf = h \frac{c}{\lambda}[/tex]

where;

c is speed of light = 3 x 10⁸ m/s

λ is the wavelength of the electromagnetic wave

Since the speed of light is constant, we can conclude that the energy of the electromagnetic wave is directly proportional to its frequency and inversely proportional to its wavelength.

Therefore, the correct option for direct proportionality is FREQUENCY

Una niña tira de un carrito por medio de una soga. Si realiza una fuerza de 10 N qué forma un ángulo de 30° con la dirección de desplazamiento. ¿Qué trabajo hace al desplazar 300cm?

Answers

Answer:

El trabajo realizado por la niña es 25.9 J.

Explanation:

El trabajo ue hace la niña viene dado por:

[tex] W = |F|*|d|*cos(\theta) [/tex]    

En donde:

F: es la fuerza realizada = 10 N

d: es el desplazamiento = 300 cm

θ: es el ángulo entre la fuerza y el desplazamiento = 30°

Entonces, el trabajo es:

[tex] W = |F|*|d|*cos(\theta) = 10 N*3 m*cos(30) = 25.9 J [/tex]    

Por lo tanto, el trabajo realizado por la niña es 25.9 J.

Espero que te sea de utilidad!        

When catching a baseball, the ball applies a force of 39.6 N to a  catcher's glove.  If the work done on the catchers glove is 47.5 J  what is the recoil (travel of)  on the mitt?

Answers

Answer:

d = 1.19 m

Explanation:

Given that,

The force applied by the ball, F = 39.6 N

The work done on the catchers glove is 47.5 J

We need to find the distance traveled by the ball. We know that,

Work done, W = Fd

Where

d is the distance traveled

[tex]d=\dfrac{W}{F}\\\\d=\dfrac{47.5 }{39.6 }\\\\d=1.19\ m[/tex]

So, it will cover 1.19 m.

An atom with a mass number of 61 amu has 33 neutrons in its nucleus.
What element is this ?

Answers

Answer:  Nickel

=========================================================

Explanation:

amu = atomic mass unit

If the amu is 61, and we have 33 neutrons, then there are 61-33 = 28 protons. The number of protons directly determines the element. Use the periodic table to find that the element Nickel (symbol "Ni") has 28 protons. The number up top shows the number of protons. For some periodic tables, the number down below (some decimal value) represents the atomic mass. This is usually the average atomic mass when considering all varieties of isotopes.

Nickel is a transition metal found in the middle portion of the periodic table, in the top-most row. It has Cobalt (Co) on the left side, Copper (Cu) on the right side, and Palladium (Pd) below it. See the diagram below.

How much force is required (in Newtons) to accelerate a 12-kg bicycle, along with its 50-kg rider, at 2 m/s2?

Give your answer as a number.

Answers

Answer:

F = 124 N

Explanation:

Given that,

The mass of bicycle and the rider, m = 12 kg + 50 kg = 62 kg

Acceleration of system, a = 2 m/s²

We need to find the force required to accelerate the system. We know that,

Net force, F = ma

Put all the values,

F = 62 kg × 2 m/s²

F = 124 N

So, the required force is equal to 124 N.

You hear a tone and recognized it to me 512 Hz and you know that the speed of sound
is 340 m/s at the surface of the earth. What is the wavelength of the sound?
Answer with three significant figures.

Answers

Answer:

Explanation:

The frequency equation is

[tex]f=\frac{v}{\lambda}[/tex] where f is frequency measured in Hz, v is velocity of sound measured in m/s and [tex]\lambda[/tex] is wavelength measured in meters. Filling in our given info:

[tex]512=\frac{340}{\lambda}[/tex] and

[tex]\lambda=\frac{340}{512}[/tex] so

[tex]\lambda=.664 m[/tex] (That is the incorrect number of sig dig's by the way. It should only be 2 since the number 340 only has 2 sig dig's)

Someone please helpppp!

Answers

i believe it’s A, i could be wrong tho 3

‏Let's say that the teacher raised the lawn bowling ball to a height of 1.6 meters before they released it . THE LAWN BOWLING BALL HAS A MASS OF 4KG . Remember that " g " ALWAYS equals 9.8 m / s² . Calculate the gravitational potential energy of the lawn bowling ball at this position

Answers

ya ok do ok bro al ya al ya la ya al

If a rod attached to the approaching charge if the rod consists of "stiff" spring-like bonds for which atoms undergo small oscillations. What can we say, about these springlike bonds when the charge is first, furthest away and second, closest to the source charge

Answers

Answer: hello options related to your question is missing attached below is the missing part of your question

answer: No charge of the length of the bonds expected because the rod did not touch the charge source ( option A )

Explanation:

When the Charge is first, Furthest away and second  and closest to the source charge. The spring like bonds can be said to have No charge of the length  of the bonds expected because the rod did not touch the charge source when Furthest away the bond with charge will be less effective

Two people are pulling straight down on ropes attached to a wheel. The wheel has a radius of 2.42 m. One person pulls in a clockwise direction with a force of 43.2 N at the end of the wheel, while the other pulls in a counterclockwise direction with a force of 67.8 N only .522 m from the center of the wheel. What is the net torque on the wheel?

Answers

Answer:

69.2 Nm

Explanation:

Since torque, τ = Fd where F = force and d = distance of force from center of wheel.

The torque due to the clockwise force F' = 43.2 N at a distance d' = radius of wheel = 2.42 m (since it is at the end of the wheel) is τ' = F'd' = 43.2 N × 2.42 m = 104.544 Nm. Since this force is counterclockwise, the torque is positive. So, τ' = 104.544 Nm

The torque due to the force F" = 67.8 N at a distance d" = 0.522 m from center of wheel is τ" = F"d" = 67.8 N × 0.522 m = 35.3916 Nm. Since this force is counterclockwise, the torque is negative. So, τ" = -35.3916 Nm

The net torque on the wheel  τ =  τ' +  τ"

= 104.544 Nm + (-35.3916 Nm)

= 104.544 Nm - 35.3916 Nm

= 69.1524 Nm

≅ 69.2 Nm

An engine is used to lift a beam weighing 9,800 N up to 290 meters.
How much work must be done to lift the beam?

Answers

Answer:

w=9800*290

w=2832000 newton

Q) A farmer moves along the boundary of a
square field of side 10 min 40 s .What will
be the magnitude of displacement of the
farmer at the end of 2 minutes 20 seconds
from his initial position?​

Answers

The answer will be 8 because kedks

Suppose object A (2 kg) is at rest and object B (10 kg) runs into it with high speed. What is the likely result?

A.
Object A will remain at rest.

B.
Object B will come to a stop.

C.
Object B's final momentum will be larger than its initial momentum.

D.
Object A will move in the direction of object B's velocity; object B will continue in that direction with a final speed less than its initial speed.

Answers

Answer:

D.

Object A will move in the direction of object B's velocity; object B will continue in that direction with a final speed less than its initial speed.

Explanation:

Conservation of Momentum. For every action will have an equal and opposite reaction. Object B will hit Object A transferring it's momentum to Object A and Object A will transfer it's momentum to Object B but since Object B is 5 times larger than Object A, Object A will only push back on Object B with a fraction of the force that Object B is pushing on Object A. Like a Bowling Ball when it hits bowling pins. The bowling ball continues to move in its direction but at a slower speed because it transferred speed to the bowling pins.

How much time in seconds did it take a tow truck using 125,000 W of power to pull a
car with 875,000 J of work?

Answers

Answer:

7 Seconds

Explanation:

P=W/t

t=W/P

t=875,000/125,000

t=7 seconds

Which sex is born with half-formed versions of
their gametes that will mature at puberty

Answers

I’m pretty sure it’s females

Answer:

females

Explanation:

The ova(egg) starts meiosis as a foetus but doesn't complete it until puberty. This is when the menstrual cycle will start to occur indicating the gametes( egg) is maturing and if not fertilized will be removed.

Two charged objects, A and B, are exerting an electric force on each other. What will happen if the charge on A is increased?

А. The charge on B will decrease.

B. The charge on B will increase.

C. The electric force between A and B will decrease.

D. The electric force between A and B will increase.​

Answers

If the charge on A increases, the force on BOTH objects will increase.

The electric force between two charges is always equal in both directions, just like the gravitational force between two masses.

So I guess choice-B  and choice-D  are both correct.  

Answer:

D. The electric force between A and B will increase.​

Explanation:

Other Questions
The following are reasons why anopinion writer should notoveranalyze the topic except whichone?A. The audience can't handle more than one opinionand analysis of the topic.B. The audience is not interested in reading aboutan idea which has been fully developed.C. The writer is allowing the audience to think moreabout the topic for themselves. Which of the following is not a compound? HCl Cl2 NaCI CO2 Complete the Steps to Solve the Equation -13x-11=-37-13x-11=-37-13x= One integer is 10 more than another, Their sum is 4. What are the 2 integers? PLEASE HELP! Choose the best word to complete the sentence. The baby boy saw ... In the mirror and started to cry. What is the name for the group of approximatley 100 cells that forms a fertilized egg PLEASE HELP QUICKLY - ATTACHED BELOW MATHS Which line is perpendicular to a line that has a slope of 1 half Find the sum of the numbers. Express your answer in scientific notation.(6.94 x 10-7)+(2.7x10-)O 7.21x 10-O 9.64 x 10-O 7.21x10-2O 9.64 x 10-7 help me help help me please The man brought three buses for 9694820. One bus cost Rs352750another us cost Rs 3029870. Find the cost of third bus. A. 6B. 2C. 0.4D. -6 What is the domain of the function on the graph What is another name for North East trade winds? 35_5634help plsI need it Which is an internal factor that may influence ones decision to have sex? Evaluate given function. Pleas helpp What interest rate is implicit in a $1,000 par value zero-coupon bond that matures in 7 years if the current price is $500. Please specify your answer in decimal terms and round your answer to the nearest thousandth (e.g., enter 12.3 percent as 0.123). How much growth or decay is 50(1.05)^t find the volume of the prism please